From 6c9b187f3a8d46ba6f8adbc338d7ea783ecf663f Mon Sep 17 00:00:00 2001 From: "W. Trevor King" Date: Fri, 1 Jun 2012 17:13:19 -0400 Subject: [PATCH] Use non-breaking space (~) between 'Figure' and the figure number. --- latex/problems/Serway_and_Jewett_8/problem02.05.tex | 2 +- latex/problems/Serway_and_Jewett_8/problem03.43.tex | 2 +- latex/problems/Serway_and_Jewett_8/problem05.25.tex | 2 +- latex/problems/Serway_and_Jewett_8/problem05.28.tex | 2 +- latex/problems/Serway_and_Jewett_8/problem05.30.tex | 2 +- latex/problems/Serway_and_Jewett_8/problem05.63.tex | 2 +- latex/problems/Serway_and_Jewett_8/problem06.21.tex | 2 +- latex/problems/Serway_and_Jewett_8/problem06.39.tex | 2 +- latex/problems/Serway_and_Jewett_8/problem08.08.tex | 2 +- latex/problems/Serway_and_Jewett_8/problem08.11.tex | 2 +- latex/problems/Serway_and_Jewett_8/problem08.15.tex | 2 +- latex/problems/Serway_and_Jewett_8/problem08.22.tex | 2 +- latex/problems/Serway_and_Jewett_8/problem09.56.tex | 6 +++--- latex/problems/Serway_and_Jewett_8/problem10.35.tex | 2 +- latex/problems/Serway_and_Jewett_8/problem10.44.tex | 2 +- latex/problems/Serway_and_Jewett_8/problem10.51.tex | 2 +- latex/problems/Serway_and_Jewett_8/problem23.17.tex | 2 +- latex/problems/Serway_and_Jewett_8/problem23.50.tex | 2 +- latex/problems/Serway_and_Jewett_8/problem23.59.tex | 2 +- latex/problems/Serway_and_Jewett_8/problem23.62.tex | 2 +- latex/problems/Serway_and_Jewett_8/problem24.11.tex | 2 +- latex/problems/Serway_and_Jewett_8/problem24.21.tex | 2 +- latex/problems/Serway_and_Jewett_8/problem24.51.tex | 2 +- latex/problems/Serway_and_Jewett_8/problem25.36.tex | 2 +- latex/problems/Serway_and_Jewett_8/problem29.40.tex | 2 +- latex/problems/Serway_and_Jewett_8/problem30.61.tex | 2 +- latex/problems/Serway_and_Jewett_8/problem30.64.tex | 2 +- 27 files changed, 29 insertions(+), 29 deletions(-) diff --git a/latex/problems/Serway_and_Jewett_8/problem02.05.tex b/latex/problems/Serway_and_Jewett_8/problem02.05.tex index 16528df..13e1894 100644 --- a/latex/problems/Serway_and_Jewett_8/problem02.05.tex +++ b/latex/problems/Serway_and_Jewett_8/problem02.05.tex @@ -1,6 +1,6 @@ \begin{problem*}{2.5} A position-time graph for a particle moving along the $x$ axis is -shown in Figure P2.5. \Part{a} Find the average velocity in the time +shown in Figure~P2.5. \Part{a} Find the average velocity in the time interval $t=1.50\U{s}$ to $t=4.00\U{s}$. \Part{b} Determine the instantaneous velocity at $t=2.00\U{s}$ by measuring the slope of the tangent to the graph. \Part{c} At what value of $t$ is the velocity diff --git a/latex/problems/Serway_and_Jewett_8/problem03.43.tex b/latex/problems/Serway_and_Jewett_8/problem03.43.tex index c3442ce..f64dd63 100644 --- a/latex/problems/Serway_and_Jewett_8/problem03.43.tex +++ b/latex/problems/Serway_and_Jewett_8/problem03.43.tex @@ -6,7 +6,7 @@ airplane is directly above you so that the vector leading from you to it is $\vect{P}_0=7.60\E{3}\jhat\U{m}$. At $t=30.0\U{s}$, the position vector leading from you to the airplane is $\vect{P}_{30}=(8.04\E{3}\ihat+7.60\E{3}\jhat)\U{m}$ as suggested in -Figure P3.43. Determine the magnitude and orientation of the +Figure~P3.43. Determine the magnitude and orientation of the airplane's position vector at $t=45.0\U{s}$. \begin{center} \begin{asy} diff --git a/latex/problems/Serway_and_Jewett_8/problem05.25.tex b/latex/problems/Serway_and_Jewett_8/problem05.25.tex index 259026c..9c76b3a 100644 --- a/latex/problems/Serway_and_Jewett_8/problem05.25.tex +++ b/latex/problems/Serway_and_Jewett_8/problem05.25.tex @@ -1,6 +1,6 @@ \begin{problem*}{5.25} A bag of cement whose weight is $F_g$ hangs in equilibrium from three -wires shown in Figure P5.24. Two of the wires make angles +wires shown in Figure~P5.24. Two of the wires make angles $\theta_1=60.0\dg$ and $\theta_2=40.0\dg$ with the horizontal. Assuming the system is in equilibrium, show that the tension in the left-hand wire is diff --git a/latex/problems/Serway_and_Jewett_8/problem05.28.tex b/latex/problems/Serway_and_Jewett_8/problem05.28.tex index e2c2867..13888a1 100644 --- a/latex/problems/Serway_and_Jewett_8/problem05.28.tex +++ b/latex/problems/Serway_and_Jewett_8/problem05.28.tex @@ -2,7 +2,7 @@ An object of mass $m_1=5.00\U{kg}$ placed on a frictionless, horizontal table is connected to a string that passes over a pulley and then is fastened to a hanging object of mass $m_2=9.00\U{kg}$ as -shown in Figure P5.28. \Part{a} Draw free-body diagrams of both +shown in Figure~P5.28. \Part{a} Draw free-body diagrams of both objects. Find \Part{b} the magnitude of that acceleration of the objects and \Part{c} the tension in the string. \begin{center} diff --git a/latex/problems/Serway_and_Jewett_8/problem05.30.tex b/latex/problems/Serway_and_Jewett_8/problem05.30.tex index 780433c..4b3e6af 100644 --- a/latex/problems/Serway_and_Jewett_8/problem05.30.tex +++ b/latex/problems/Serway_and_Jewett_8/problem05.30.tex @@ -1,6 +1,6 @@ \begin{problem*}{5.30} Two objects are connected by a light string that passes over a -frictionless pulley as shown in Figure P5.30. Assume the incline is +frictionless pulley as shown in Figure~P5.30. Assume the incline is frictionless and take $m_1=2.00\U{kg}$, $m_2=6.00\U{kg}$, and $\theta=55.0\dg$. \Part{a} Draw free-body diagrams of both objects. Find \Part{b} the magnitude of the acceleration of the diff --git a/latex/problems/Serway_and_Jewett_8/problem05.63.tex b/latex/problems/Serway_and_Jewett_8/problem05.63.tex index bccafb2..3a79272 100644 --- a/latex/problems/Serway_and_Jewett_8/problem05.63.tex +++ b/latex/problems/Serway_and_Jewett_8/problem05.63.tex @@ -1,6 +1,6 @@ \begin{problem*}{5.63} A crate of wieght $F_g$ is pushed by a force $\vect{P}$ on a -horizontal floor as shown in Figure P5.63. The coefficient of static +horizontal floor as shown in Figure~P5.63. The coefficient of static friction is $\mu_s$, and $\vect{P}$ is directed at an angle $\theta$ below the horizontal. \Part{a} Show that the minimum value of $P$ that will move the crate is given by diff --git a/latex/problems/Serway_and_Jewett_8/problem06.21.tex b/latex/problems/Serway_and_Jewett_8/problem06.21.tex index a84a5e6..f702503 100644 --- a/latex/problems/Serway_and_Jewett_8/problem06.21.tex +++ b/latex/problems/Serway_and_Jewett_8/problem06.21.tex @@ -1,6 +1,6 @@ \begin{problem*}{6.21} An object of mass $m=0.500\U{kg}$ is suspended from the ceiling of an -accelertating truck as shown in Figure P6.21. Taking +accelertating truck as shown in Figure~P6.21. Taking $a=3.00\U{m/s$^2$}$, find \Part{a} the angle $\theta$ that the string makes with the vertical and \Part{b} the tension $T$ in the string. \begin{center} diff --git a/latex/problems/Serway_and_Jewett_8/problem06.39.tex b/latex/problems/Serway_and_Jewett_8/problem06.39.tex index afe642b..095c2ed 100644 --- a/latex/problems/Serway_and_Jewett_8/problem06.39.tex +++ b/latex/problems/Serway_and_Jewett_8/problem06.39.tex @@ -1,7 +1,7 @@ \begin{problem*}{6.39} A string under a tension of $50.0\U{N}$ is used to whirl a rock in a horizontal circle of radius $2.50\U{m}$ at a speed of $20.4\U{m/s}$ on -a frictionless surface as shown in Figure P.39. As the string is +a frictionless surface as shown in Figure~P.39. As the string is pulled in, the speed of the rock increases. When the string is $1.00\U{m/s}$ long and the speed of the rock is $51.0\U{m/s}$, the string breaks. What is the breaking strength, in newtons, of the diff --git a/latex/problems/Serway_and_Jewett_8/problem08.08.tex b/latex/problems/Serway_and_Jewett_8/problem08.08.tex index 7f993ac..dca358a 100644 --- a/latex/problems/Serway_and_Jewett_8/problem08.08.tex +++ b/latex/problems/Serway_and_Jewett_8/problem08.08.tex @@ -1,6 +1,6 @@ \begin{problem*}{8.8} Two objects are connected by a light string passing over a light, -frictionless pulley as shown in Figure P8.7. The object of mass $m_1$ +frictionless pulley as shown in Figure~P8.7. The object of mass $m_1$ is released from rest at a height $h$ above the table. Using the isolated system model, \Part{a} determine the speed of $m_2$ just as $m_1$ hits the table and \Part{b} find the maximum height above the diff --git a/latex/problems/Serway_and_Jewett_8/problem08.11.tex b/latex/problems/Serway_and_Jewett_8/problem08.11.tex index b66eabb..01ce896 100644 --- a/latex/problems/Serway_and_Jewett_8/problem08.11.tex +++ b/latex/problems/Serway_and_Jewett_8/problem08.11.tex @@ -1,5 +1,5 @@ \begin{problem*}{8.11} -The system shown in Figure P8.11 consists of a light, inextensible +The system shown in Figure~P8.11 consists of a light, inextensible cord, light, frictionless pulleys, and blocks of equal mass. Notice that block $B$ is attached to one of the pulleys. The system is initially held at rest so that the blocks are at the same height above diff --git a/latex/problems/Serway_and_Jewett_8/problem08.15.tex b/latex/problems/Serway_and_Jewett_8/problem08.15.tex index c6dd81e..37c3f64 100644 --- a/latex/problems/Serway_and_Jewett_8/problem08.15.tex +++ b/latex/problems/Serway_and_Jewett_8/problem08.15.tex @@ -1,6 +1,6 @@ \begin{problem*}{8.15} A block of mass $m=2.00\U{kg}$ is attached to a spring of force -constant $k=500\U{N/m}$ as shown in Figure P8.15. The block is pulled +constant $k=500\U{N/m}$ as shown in Figure~P8.15. The block is pulled to a position $x_i=5.00\U{cm}$ to the right of equilibrium and released from rest. Find the speed the block has as it passes through equilibrium if \Part{a} the horizontal surface is frictionless diff --git a/latex/problems/Serway_and_Jewett_8/problem08.22.tex b/latex/problems/Serway_and_Jewett_8/problem08.22.tex index df0f2a8..8d37f24 100644 --- a/latex/problems/Serway_and_Jewett_8/problem08.22.tex +++ b/latex/problems/Serway_and_Jewett_8/problem08.22.tex @@ -1,6 +1,6 @@ \begin{problem*}{8.22} The coefficient of friction between the block of mass $m_1=3.00\U{kg}$ -and the surface in Figure P8.22 is $\mu_s=0.400$. The system starts +and the surface in Figure~P8.22 is $\mu_s=0.400$. The system starts from rest. What is the speed of the bal of mass $m_2=5.00\U{kg}$ when it has fallen a distance $h=1.50\U{m}$? % m1-block-on-table -- pulley -- hanging m2 diff --git a/latex/problems/Serway_and_Jewett_8/problem09.56.tex b/latex/problems/Serway_and_Jewett_8/problem09.56.tex index 818e052..9992a4b 100644 --- a/latex/problems/Serway_and_Jewett_8/problem09.56.tex +++ b/latex/problems/Serway_and_Jewett_8/problem09.56.tex @@ -1,9 +1,9 @@ \begin{problem*}{9.56} -Figure P9.56 shows three points in the operation of the ballistic +Figure~P9.56 shows three points in the operation of the ballistic pendulum discussed in Example 9.6 (and shown in Fig.~9.9b). The -projectile approaches the pendulum in Figure P9.56a. Figure P9.56b +projectile approaches the pendulum in Figure~P9.56a. Figure~P9.56b shows the situation just after the projectile is captured in the -pendulum. In Figure P9.56c, the pendulum arm has swung upward and +pendulum. In Figure~P9.56c, the pendulum arm has swung upward and come to rest at a height $h$ above its initial position. \Part{a} Prove that the ratio of kinetic energy of the projectile-pendulum system immediately after the collision to the kenetic energy diff --git a/latex/problems/Serway_and_Jewett_8/problem10.35.tex b/latex/problems/Serway_and_Jewett_8/problem10.35.tex index 7993788..4fa6891 100644 --- a/latex/problems/Serway_and_Jewett_8/problem10.35.tex +++ b/latex/problems/Serway_and_Jewett_8/problem10.35.tex @@ -1,5 +1,5 @@ \begin{problem*}{10.35} -Find the net torque on the wheel in Figure P10.35 about the axle +Find the net torque on the wheel in Figure~P10.35 about the axle through $O$, taking $a=10.0\U{cm}$ and $b=25.0\U{cm}$. % 10.0N at a point b N of O, pulling E % 9.00N at a point b E of O, pulling S diff --git a/latex/problems/Serway_and_Jewett_8/problem10.44.tex b/latex/problems/Serway_and_Jewett_8/problem10.44.tex index 42170e6..0223ca3 100644 --- a/latex/problems/Serway_and_Jewett_8/problem10.44.tex +++ b/latex/problems/Serway_and_Jewett_8/problem10.44.tex @@ -1,5 +1,5 @@ \begin{problem*}{10.44} -Consider the system shown in Figure P10.44 with $m_1=20.0\U{kg}$, +Consider the system shown in Figure~P10.44 with $m_1=20.0\U{kg}$, $m_2=12.5\U{kg}$, $R=0.200\U{m}$, and the mass of the pulley $M=5.00\U{kg}$. Object $m_2$ is resting on the floor, and object $m_1$ is $4.00\U{m}$ above the floor when it is released from rest. diff --git a/latex/problems/Serway_and_Jewett_8/problem10.51.tex b/latex/problems/Serway_and_Jewett_8/problem10.51.tex index ae5dde8..7dcb65b 100644 --- a/latex/problems/Serway_and_Jewett_8/problem10.51.tex +++ b/latex/problems/Serway_and_Jewett_8/problem10.51.tex @@ -3,7 +3,7 @@ An object with a mass $m=5.10\U{kg}$ is attached to the free end of a light string wrapped around a reel of radius $R=0.250\U{m}$ and mass $M=3.00\U{kg}$. The reel is a solid disk, free to rotate in a vertical plane about the horiizontal axis passing through its center -as shown in Figure P10.51. The suspended object is released from rest +as shown in Figure~P10.51. The suspended object is released from rest $6.00\U{m}$ above the floor. Determine \Part{a} the tension in the string, \Part{b} the acceleration of the object, and \Part{c} the speed with which the object hits the floor. \Part{d} Verify your diff --git a/latex/problems/Serway_and_Jewett_8/problem23.17.tex b/latex/problems/Serway_and_Jewett_8/problem23.17.tex index 55b05bb..435b0bc 100644 --- a/latex/problems/Serway_and_Jewett_8/problem23.17.tex +++ b/latex/problems/Serway_and_Jewett_8/problem23.17.tex @@ -1,6 +1,6 @@ \begin{problem*}{23.17} A point charge $+2Q$ is at the origin and a point charge $-Q$ is -located along the $x$ axis at $x=d$ as in Figure P23.17. Find a +located along the $x$ axis at $x=d$ as in Figure~P23.17. Find a symbolic expression for the net force on a third point charge $+Q$ located along the $y$ axis at $y=d$. \begin{center} diff --git a/latex/problems/Serway_and_Jewett_8/problem23.50.tex b/latex/problems/Serway_and_Jewett_8/problem23.50.tex index 37c01a5..1c3d0b8 100644 --- a/latex/problems/Serway_and_Jewett_8/problem23.50.tex +++ b/latex/problems/Serway_and_Jewett_8/problem23.50.tex @@ -1,6 +1,6 @@ \begin{problem*}{23.50} A small sphere of charge $q_1=0.800\U{$\mu$C}$ hangs from the end of a -spring as in Figure P23.50a. When another small sphere of charge +spring as in Figure~P23.50a. When another small sphere of charge $q_2=-0.600\U{$\mu$C}$ is held beneath the first sphere as in Figure P23.50b, the spring stretches by $d=3.50\U{cm}$ from its original length and reaches a new equilibrium position with a separation diff --git a/latex/problems/Serway_and_Jewett_8/problem23.59.tex b/latex/problems/Serway_and_Jewett_8/problem23.59.tex index e5d9df9..5c5a8e0 100644 --- a/latex/problems/Serway_and_Jewett_8/problem23.59.tex +++ b/latex/problems/Serway_and_Jewett_8/problem23.59.tex @@ -1,6 +1,6 @@ \begin{problem*}{23.59} A charged cork ball of mass $1.00\U{g}$ is suspended on a light string -in the presence of a uniform electric field as shown in Figure P23.59. +in the presence of a uniform electric field as shown in Figure~P23.59. When $\vect{E}=(3.00\ihat+5.00\jhat)\E{5}\U{N/C}$, the ball is in equilibrium at $\theta=37.0\dg$. Find \Part{a} the charge on the ball and \Part{b} the tension in the string. diff --git a/latex/problems/Serway_and_Jewett_8/problem23.62.tex b/latex/problems/Serway_and_Jewett_8/problem23.62.tex index eb338d3..a3e67aa 100644 --- a/latex/problems/Serway_and_Jewett_8/problem23.62.tex +++ b/latex/problems/Serway_and_Jewett_8/problem23.62.tex @@ -1,6 +1,6 @@ \begin{problem*}{23.62} Four identical charged particles ($q=+10.0\U{$\mu$C}$) are located on -the corners of a rectangle as shown in Figure P23.62. The dimensions +the corners of a rectangle as shown in Figure~P23.62. The dimensions of the rectangle are $L=60.0\U{cm}$ and $W=15.0\U{cm}$. Calculate \Part{a} the magnitude and \Part{b} the direction of the total electric force exerted on the charge at the lower left corner by diff --git a/latex/problems/Serway_and_Jewett_8/problem24.11.tex b/latex/problems/Serway_and_Jewett_8/problem24.11.tex index 0da2b88..8194132 100644 --- a/latex/problems/Serway_and_Jewett_8/problem24.11.tex +++ b/latex/problems/Serway_and_Jewett_8/problem24.11.tex @@ -1,6 +1,6 @@ \begin{problem*}{24.11} Four closed surfaces, $S_1$ through $S_4$, together with the charges -$-2Q$, $Q$, and $-Q$ are sketched in Figure P24.11. (The colored +$-2Q$, $Q$, and $-Q$ are sketched in Figure~P24.11. (The colored lines are the intersections of the surfaces with the page.) Find the electric flux through each surface. % -2Q diff --git a/latex/problems/Serway_and_Jewett_8/problem24.21.tex b/latex/problems/Serway_and_Jewett_8/problem24.21.tex index 1741cf8..ea4ce5d 100644 --- a/latex/problems/Serway_and_Jewett_8/problem24.21.tex +++ b/latex/problems/Serway_and_Jewett_8/problem24.21.tex @@ -1,5 +1,5 @@ \begin{problem*}{24.21} -Figure P24.21 represents the top view of a cubic gaussian surface in a +Figure~P24.21 represents the top view of a cubic gaussian surface in a uniform electric field \vect{E} oriented parallel to the top and bottom faces of the cube. The field makes an angle $\theta$ with side $a$, and the area of each face is $A$. In sumbolic form, find the diff --git a/latex/problems/Serway_and_Jewett_8/problem24.51.tex b/latex/problems/Serway_and_Jewett_8/problem24.51.tex index 80c883d..62065fe 100644 --- a/latex/problems/Serway_and_Jewett_8/problem24.51.tex +++ b/latex/problems/Serway_and_Jewett_8/problem24.51.tex @@ -3,7 +3,7 @@ A solid insulating sphere of radius $a=5.00\U{cm}$ carries a net positive charge of $Q=3.00\U{$\mu$C}$ uniformly distributed throughout its volume. Concentric with this sphere is a conducting spherical shell with inner radius $b=10.0\U{cm}$ and outer radius $c=15.0\U{cm}$ -as shown in Figure P24.51, having net charge $q=-1.00\U{$\mu$C}$. +as shown in Figure~P24.51, having net charge $q=-1.00\U{$\mu$C}$. Prepare a graph of the magnitude of the electric field due to this configuration versus $r$ for $0